rational number 3 by 40 is equals to​

Answers

Answer 1

Answer:

6/80, 9/120, 12/160 etc

Answer 2

Answer:

3/40 = 6/80 = 9/120 = 12/160 etc......

Hope it helps

Mark it as Brainliest pls!!!!! ( the crown icon)


Related Questions

Given the following three points, find by hand the quadratic function they represent.
(-1,-8), (0, -1),(1,2)
(1 point)
Of(x) = -51% + 87 - 1
O f(x) = -3.2? + 4.1 - 1
Of(t) = -202 + 5x - 1
Of(1) = -3.1? + 10.1 - 1​

Answers

Answer:

The correct option is;

f(x) = -2·x² + 5·x - 1

Step-by-step explanation:

Given the points

(-1, -8), (0, -1), (1, 2), we have;

The general quadratic function;

f(x) = a·x² + b·x + c

From the given points, when x = -1, y = -8, which gives

-8 = a·(-1)² + b·(-1) + c = a - b + c

-8 =  a - b + c.....................................(1)

When x = 0, y = -1, which gives;

-1 = a·0² + b·0 + c = c

c = -1.....................................................(2)

When x = 1, y = 2, which gives;

2 = a·1² + b·1 + c = a + b + c...............(3)

Adding equation (1) to (3), gives;

-8 + 2 = a - b + c + a + b + c

-6 = 2·a + 2·c

From equation (2), c = -1, therefore;

-6 = 2·a + 2×(-1)

-2·a  = 2×(-1)+6 = -2 + 6 = 4

-2·a = 4

a = 4/-2 = -2

a = -2

From equation (1), we have;

-8 =  a - b + c = -2 - b - 1 = -3 - b

-8 + 3 = -b

-5 = -b

b = 5

The equation is therefore;

f(x) = -2·x² + 5·x - 1

The correct option is f(x) = -2·x² + 5·x - 1.

prove tan(theta/2)=sin theta/1+cos theta for theta in quadrant 1 by filling in the calculations and reasons. PLEASE HELP!!!!

Answers

Answer:

See explanation

Step-by-step explanation:

We have to prove the identity

[tex]tan(\frac{\Theta }{2})=\frac{sin\Theta}{1+cos\Theta }[/tex]

We will take right hand side of the identity

[tex]\frac{sin\Theta}{1+cos\Theta}=\frac{2sin(\frac{\Theta }{2})cos(\frac{\Theta }{2})}{1+[2cos^{2}(\frac{\Theta }{2})-1]}[/tex]

[tex]=\frac{2sin(\frac{\Theta }{2})cos(\frac{\Theta }{2})}{2cos^{2}(\frac{\Theta }{2})}=\frac{sin(\frac{\Theta }{2})}{cos(\frac{\Theta }{2})}[/tex]

[tex]=tan(\frac{\Theta }{2})[/tex] [ Tan θ will be positive since θ lies in 1st quadrant ]

PLEaSE HELP!!!!!! will give brainliest to first answer

Answers

Answer:

The coordinates of A'C'S'T' are;

A'(-7, 2)

C'(-9, -1)

S'(-7, -4)

T'(-5, -1)

The correct option is;

B

Step-by-step explanation:

The coordinates of the given quadrilateral are;

A(-3, 1)

C(-5, -2)

S(-3, -5)

T(-1, -2)

The required transformation is T₍₋₄, ₁₎ which is equivalent to a movement of 4 units in the leftward direction and 1 unit upward

Therefore, we have;

A(-3, 1) + T₍₋₄, ₁₎ = A'(-7, 2)

C(-5, -2) + T₍₋₄, ₁₎ = C'(-9, -1)

S(-3, -5) + T₍₋₄, ₁₎ = S'(-7, -4)

T(-1, -2) + T₍₋₄, ₁₎ = T'(-5, -1)

Therefore, the correct option is B

the cube root of 2 to the seventh power

Answers

Answer:

4 2^(1/3) or 5.0396841995794926590688424291129134022810058588060319203279004486... decimal

Step-by-step explanation:

Simplify the following:

(2^(1/3))^7

Hint: | For all a>=0, (a^(1/3))^m = a^(m/3). Apply this to (2^(1/3))^7.

Multiply exponents. (2^(1/3))^7 = 2^(7/3):

2^(7/3)

Hint: | Separate the exponent of 2^(7/3) into integer and fractional parts.

2^(7/3) = 2^(6/3 + 1/3) = 2^(6/3)×2^(1/3):

2^(6/3) 2^(1/3)

Hint: | Divide 6 by 3.

6/3 = (3×2)/3 = 2:

2^2 2^(1/3)

Hint: | Evaluate 2^2.

2^2 = 4:

Answer:  4 2^(1/3) or 5.0396841995794926590688424291129134022810058588060319203279004486... decimal

Which of the following best describes the graph shown below?
16
A1
1
14
O A This is the graph of a linear function
B. This is the graph of a one-to-one function
C. This is the graph of a function, but it is not one to one
D. This is not the graph of a function

Answers

Answer: C. It is a function, but is it not one-to-one

The vertical line test helps us see that we have a function. Note how it is not possible to draw a single straight line through more than one point on the curve. Any x input leads to exactly one y output. This graph passes the vertical line test. Therefore it is a function.

The function is not one-to-one because the graph fails the horizontal line test. Here it is possible to draw a single straight horizontal line through more than one point on the curve. The horizontal line through y = 2 is one example of many where the graph fails the horizontal line test, meaning the function is not one-to-one.

The term "one-to-one" means that each y value only pairs up with one x value. Here we have something like y = 2 pair up with multiple x values at the same time. This concept is useful when it comes to determining inverse functions.

Shaquira is baking cookies to put in packages for a fundraiser. Shaquira has made 86 8686 chocolate chip cookies and 42 4242 sugar cookies. Shaquira wants to create identical packages of cookies to sell, and she must use all of the cookies. What is the greatest number of identical packages that Shaquira can make?

Answers

Answer: 2

Step-by-step explanation:

Given: Shaquira has made 86  chocolate chip cookies and 42 sugar cookies.

Shaquira wants to create identical packages of cookies to sell, and she must use all of the cookies.

Now, the greatest number of identical packages that Shaquira can make= GCD of 86 and 42

Prime factorization of 86 and 42:

86 = 2 ×43

42 = 2 × 3 × 7

GCD of 86 and 42 = 2   [GCD = greatest common factor]

Hence, the greatest number of identical packages that Shaquira can make =2

What is the value of b?

Answers

Answer:

  55°

Step-by-step explanation:

Perhaps you want the measure of angle B. (There is no "b" in the figure.)

That measure is half the measure of the intercepted arc:

  m∠B = 110°/2 = 55°

Angle B is 55°.

Given the equations of a straight line f(x) (in slope-intercept form) and a parabola g(x) (in standard form), describe how to determine the number of intersection points, without finding the coordinates of such points. Do not give an example.

Answers

Answer:

Step-by-step explanation:

Hello, when you try to find the intersection point(s) you need to solve a system like this one

[tex]\begin{cases} y&= m * x + p }\\ y &= a*x^2 +b*x+c }\end{cases}[/tex]

So, you come up with a polynomial equation like.

[tex]ax^2+bx+c=mx+p\\\\ax^2+(b-m)x+c-p=0[/tex]

And then, we can estimate the discriminant.

[tex]\Delta=(b-m)^2-4*a*(c-p)[/tex]

If [tex]\Delta<0[/tex] there is no real solution, no intersection point.

If [tex]\Delta=0[/tex] there is one intersection point.

If [tex]\Delta>0[/tex] there are two real solutions, so two intersection points.

Hope this helps.

A spinner has 3 red spaces, 5 white spaces, and 1 black space. If the spinner is
spun once, what is the theoretical probability of the spinner NOT stopping on
red?
P(Not red) =

Answers

Answer:

[tex]\frac{2}{3}[/tex]

Step-by-step explanation:

If we have 3 red spaces, 5 white spaces, and one blank space, there are a total of 9 spaces.

Since there are 3 red spaces, there is a [tex]\frac{3}{9} = \frac{1}{3}[/tex] chance of getting a red. However, the question asks the probability of not getting a red, so the chances of not getting a red are [tex]1 -\frac{1}{3} = \frac{2}{3}[/tex].

Hope this helped!

Given that p=x^2-y^2/x^2+xy
I. Express p in the simplest form
ii. Find the value of p, if x=-4 and y=-6

Answers

Answer:

When x = -4 and y = -6, p = 37.75

Step-by-step explanation:

Given that p = x² - y²/x² + x·y, we have;

p = (x² × x² -y² + x·y×x²)/x²

p = (x²⁺² - y² + x¹⁺² × y)/x²

p = (x⁴ - y² + x³·y)/x²

Therefore, p in the simplest form is given as follows;

[tex]p = \dfrac{x^4 - y^2 + x^3 \cdot y }{x^2}[/tex]

To find the value of p when x = -4 and y = -6, we plug in the value of x and y into the above equation to get the following equation;

[tex]p = \dfrac{(-4)^4 - (-6)^2 + (-4)^3 \cdot (-6) }{(-4)^2} = 37.75[/tex]

Therefore, the value of p when x = -4 and y = -6 is equal to 37.75.

20. A pool holds 1440 cubic feet of water, the city charges $1.75 per cubic meter of water used.
How much will it cost to fill the pool?

Answers

Answer:Conversion units

Step-by-step explanation: 1 ft^3= 0.028m^3 .: 1440ft^3=40.776m^3, so $1.75x40.776=$71.358~ $71.36.:

Answer:

$71.36

Step-by-step explanation:

1 foot = 0.3048 metros

1 cubic feet = (0.3048metros)³ = 0.02932 cubic meters   (aprox.)

1440 cubic feet = 1440*0.02932 = 40.7763 m

$1.75 por cubic meter:

1.75*40.7763 = $71.36

how do you solve 2m-10=44+8m

Answers

Answer:

m = -9

Step-by-step explanation:

2m-10=44+8m

Subtract 2m from each side

2m-2m-10=44+8m-2m

-10 = 44+6m

Subtract 44 from each side

-10-44 = 44-44+6m

-54 = 6m

Divide by 6

-54/6 = 6m/6

-9 = m

Answer:

solve by solving the salvation for equation don't be a slave get educated from what's gave

PLEASE HELP
Find the area and the perimeter of the shaded regions below. Give your answer as a completely simplified exact value in terms of π (no approximations). The figures below are based on semicircles or quarter circles and problems b), c), and d) are involving portions of a square.

Answers

Answer:

perimeter is  4 sqrt(29) + 4pi  cm

area is 40 + 8pi cm^2

Step-by-step explanation:

We have a semicircle and a triangle

First the semicircle with diameter 8

A = 1/2 pi r^2 for a semicircle

r = d/2 = 8/2 =4

A = 1/2 pi ( 4)^2

  =1/2 pi *16

  = 8pi

Now the triangle with base 8 and height 10

A = 1/2 bh

  =1/2 8*10

  = 40

Add the areas together

A = 40 + 8pi cm^2

Now the perimeter

We have 1/2 of the circumference

1/2 C =1/2 pi *d

         = 1/2 pi 8

        = 4pi

Now we need to find the length of the hypotenuse of the right triangles

using the pythagorean theorem

a^2+b^2 = c^2

The base is 4 ( 1/2 of the diameter) and the height is 10

4^2 + 10 ^2 = c^2

16 + 100 = c^2

116 = c^2

sqrt(116) = c

2 sqrt(29) = c

Each hypotenuse is the same so we have

hypotenuse + hypotenuse + 1/2 circumference

2 sqrt(29) + 2 sqrt(29) + 4 pi

4 sqrt(29) + 4pi  cm

Step-by-step explanation:

First we need to deal with the half circle. The radius of this circle is 4, because the diameter is 8. The formula for the circumference of a circle is 2piR.

2pi4 so the perimeter for the half circle would be 8pi/2.

The area of that half circle would be piR^2 so 16pi/2.

Now moving on the triangle part, we need to find the hypotenuse side of AC. We will use the pythagoram theorem. 4^2+10^2=C^2

16+100=C^2

116=C^2

C=sqrt(116)

making the perimeter of this triangle 2×sqrt(116)

The area of this triangle is 8×10=80, than divided by 2 which is equal to 40.

We than just need to add up the perimeters and areas for both the half circle and triangle.

The area would be equal to 8pi+40

The perimeter would be equal to 4pi+4(sqrt(29))

Cam’s tent (shown below) is a triangular prism.
Find the surface are, including the floor of his tent
PLEASE HELP

Answers

Answer:

21.4 m²

Step-by-step explanation:

To find the surface area of this whole triangular prism, we have to look at the bases (the triangles), find their surface area, then look at the sides (the rectangles) and find theirs.

Let's start with the triangles. The area of any triangle is [tex]\frac{bh}{2}[/tex]. The base of this triangle is 2m (because there are 2 one meters) and the height is 1.7m.

[tex]\frac{2\cdot1.7}{2} = \frac{3.4}{2} = 1.7[/tex]

So the area of one of these triangles is 1.7m. Multiplying this by two, because there are two triangles in this prism:

[tex]1.7\cdot2=3.4[/tex]

Now let's find the area of the sides.

The side lengths are 2 and 3, so

[tex]2\cdot3=6[/tex], and there are 3 sides (including the bottom/floor) so [tex]6\cdot3=18[/tex].

Now we add.

[tex]18+3.4=21.4[/tex] m².

Hope this helped!

Answer: 21.4 square meters^2

Step-by-step explanation:

ajhejskxnkrlsncjeksuf9

Answers

Yes, agreed, how are you doing? Good? Great!!

Find the vertex of f(x)= x^2+ 6x + 36


Pls help soon

Answers

Answer:

vertex(-3,27)

Step-by-step explanation:

f(x)= x^2+ 6x + 36 ( a=1,b=6,c=36)

V(h,k)

h=-b/2a=-6/2=-3

k=f(-3)=3²+6(-3)+36

f(-3)=9-18+36=27

vertex(-3,27)

NEED ASAP What is the quotient and remainder of 8,595 ÷ 24?

Answers

Answer:

358.125

Step-by-step explanation:

Answer:

358 3/24

Step-by-step explanation:  

1. Suzette ran and biked for a total of 80 miles in 9 hours. Her average running speed was 5 miles per hour (mph) and her average biking speed was 12 mph. Let x = total hours Suzette ran. Let y = total hours Suzette biked. Use substitution to solve for x and y. Show your work. Check your solution. (a) How many hours did Suzette run? (b) How many hours did she bike?

Answers

Answer:

a) Suzette ran for 4 hours

b) Suzette biked for 5 hours

Step-by-step explanation:

Speed is rate of distance traveled, it is the ratio of distance traveled to time taken. It is given by:

Speed = distance / time

The total distance ran and biked by Suzette (d) = 80 miles, while the total time ran and biked by Suzette (t) = 9 hours.

For running:

Her speed was 5 miles per hour, let the total hours Suzette ran be x and the total distance she ran be p, hence since Speed = distance / time, therefore:

5 = p / x

p = 5x

For biking:

Her speed was 12 miles per hour, let the total hours Suzette ran be y and the total distance she ran be q, hence since Speed = distance / time, therefore:

12 = q / y

q = 12y

The total distance ran and biked by Suzette (d) = Distance biked + distance ran

d = p + q

80 = p + q

80 = 5x + 12y                 (1)

The total time taken to run and bike by Suzette (t) = time spent to bike + time spent to run

t = x + y

9 = x + y                         (2)

Solving equation 1 and equation 2, multiply equation 2 by 5 and subtract from equation 1:

7y = 35

y = 35/7

y = 5 hours

Put y = 5 in equation 2:

9 = x + 5

x = 9 -5

x = 4 hours

a) Suzette ran for 4 hours

b) Suzette biked for 5 hours

Consider the following system of equations: y=2x−2 6x+3y=2 The graph of these equations consists of two lines that: 1. intersect at more than one point. 2. intersect in an infinite number of points. 3. intersect at exactly one point. 4. do not intersect.

Answers

Answer:

3.  Intersect at exactly one point.  ( (2/3), (-2/3) )

Step-by-step explanation:

To make the comparison of these lines easier, let's rewrite the 2nd equation into slope-intercept form, as the 1st equation is in slope-intercept form.

[1] y = 2x - 2

---------------------

[2] 6x + 3y = 2 ==> 3y = 2 - 6x ==> y = -2x + (2/3)

[2] y = -2x + (2/3)

So now that we have both equations in slope-intercept form, we can see that the two equations are both linear, have different slopes, and have different y-intercepts.

Since these equations have both different slopes and different y-intercepts, we know that the lines will cross at least one point.  We can confirm that the lines only cross at a single point using the fact that both equations are linear, meaning there will only be one point of crossing.  To find that point, we can simply set the equations equal to each other.

y = 2x - 2

y = -2x + (2/3)

2x - 2 = -2x + (2/3)

4x = (8/3)

x = (8/12) = (2/3)

And plug this x value back into one of the equations:

y = 2x - 2

y = 2(2/3) - 2

y = (4/3) - (6/3)

y = (-2/3)

Thus these lines only cross at the point ( (2/3), (-2/3) ).

Cheers.

Answer:

I don't understand the question

What the relation of 1/c=1/c1+1/c2 hence find c​

Answers

[tex]\frac 1c=\frac1{c_1}+\frac1{c_2} [/tex]

$\frac1c=\frac{c_1+c_2}{c_1c_2}$

$\implies c=\frac{c_1c_2}{c_1+c_2}$

I need help fast please

Answers

Answer:

Difference : 4th option

Step-by-step explanation:

The first thing we want to do here is to factor the expression x² + 3x + 2. This will help us if it is similar to the factored expression " ( x + 2 )( x + 1 ). " The denominators will be the same, and hence we can combine the fractions.

x² + 3x + 2 - Break the expression into groups,

( x² + x ) + ( 2x + 2 ) - Factor x from x² + x and 2 from 2x + 2,

x( x + 1 ) + 2( x + 2 ) - Group,

( x + 2 )( x + 1 )

This is the same as the denominator of the other fraction, and therefore we can combine the fractions.

x - 1 / ( x + 2 )( x + 1 )

As you can see this is not any of the options present, as we have not expanded ( x + 2 )( x + 1 ). Remember previously that ( x + 2 )( x + 1 ) = x² + 3x + 2. Hence our solution is x - 1 / x² + 3x + 2, or option d.

I need hellp please its my last chance to become a senior please someone

Answers

Answer:

d= 6

r= 6/2

r=3

V= π. r². h

V= π . 3². 14

V= π. 9 . 14

V= π 126 cm³

V= 126 π cm³ (π not in number)

hope it helps^°^

Answer:if you use the formula it is 126 pi cm cubed

The answer is c

Step-by-step explanation:

Use the discriminant to determine the number of real solutions to the equation. −8m^2+2m=0

Answers

Answer:

discriminant is b²-4ac

= 2²-4(-8)(0)

= 0

one solution

hope this helps :)

Complete the square to transform the expression x2 - 2x - 2 into the form a(x - h)2 + k

Answers

Answer:

A

Step-by-step explanation:

Find the vertex form of the quadratic function below.

y = x^2 - 4x + 3

This quadratic equation is in the form y = a{x^2} + bx + cy=ax  

2

+bx+c. However, I need to rewrite it using some algebraic steps in order to make it look like this…

y = a(x - h)^2 + k

This is the vertex form of the quadratic function where \left( {h,k} \right)(h,k) is the vertex or the “center” of the quadratic function or the parabola.

Before I start, I realize that a = 1a=1. Therefore, I can immediately apply the “completing the square” steps.

STEP 1: Identify the coefficient of the linear term of the quadratic function. That is the number attached to the xx-term.

STEP 2: I will take that number, divide it by 22 and square it (or raise to the power 22).

STEP 3: The output in step #2 will be added and subtracted on the same side of the equation to keep it balanced.

Think About It: If I add 44 on the right side of the equation, then I am technically changing the original meaning of the equation. So to keep it unchanged, I must subtract the same value that I added on the same side of the equation.

STEP 4: Now, express the trinomial inside the parenthesis as a square of a binomial, and simplify the outside constants.

After simplifying, it is now in the vertex form y = a{\left( {x - h} \right)^2} + ky=a(x−h)  

2

+k where the vertex \left( {h,k} \right)(h,k) is \left( {2, - 1} \right)(2,−1).

Visually, the graph of this quadratic function is a parabola with a minimum at the point \left( {2, - 1} \right)(2,−1). Since the value of “aa” is positive, a = 1a=1, then the parabola opens in upward direction.

Example 2: Find the vertex form of the quadratic function below.

The approach to this problem is slightly different because the value of “aa” does not equal to 11, a \ne 1a  

​  

=1. The first step is to factor out the coefficient 22 between the terms with xx-variables only.

STEP 1: Factor out 22 only to the terms with variable xx.

STEP 2: Identify the coefficient of the xx-term or linear term.

STEP 3: Take that number, divide it by 22, and square.

STEP 4: Now, I will take the output {9 \over 4}  

4

9

​  

 and add it inside the parenthesis.

By adding {9 \over 4}  

4

9

​  

 inside the parenthesis, I am actually adding 2\left( {{9 \over 4}} \right) = {9 \over 2}2(  

4

9

​  

)=  

2

9

​  

 to the entire equation.

Why multiply by 22 to get the “true” value added to the entire equation? Remember, I factored out 22 in the beginning. So for us to find the real value added to the entire equation, we need to multiply the number added inside the parenthesis by the number that was factored out.

STEP 5: Since I added {9 \over 2}  

2

9

​  

 to the equation, then I should subtract the entire equation by {9 \over 2}  

2

9

​  

 also to compensate for it.

STEP 6: Finally, express the trinomial inside the parenthesis as the square of binomial and then simplify the outside constants. Be careful combining the fractions.

It is now in the vertex form y = a{\left( {x - h} \right)^2} + ky=a(x−h)  

2

+k where the vertex \left( {h,k} \right)(h,k) is \left( {{{ - \,3} \over 2},{{ - 11} \over 2}} \right)(  

2

−3

​  

,  

2

−11

​  

).

Example 3: Find the vertex form of the quadratic function below.

Solution:

Factor out - \,3−3 among the xx-terms.

The coefficient of the linear term inside the parenthesis is - \,1−1. Divide it by 22 and square it. Add that value inside the parenthesis. Now, figure out how to make the original equation the same. Since we added {1 \over 4}  

4

1

​  

 inside the parenthesis and we factored out - \,3−3 in the beginning, that means - \,3\left( {{1 \over 4}} \right) = {{ - \,3} \over 4}−3(  

4

1

​  

)=  

4

−3

​  

 is the value that we subtracted from the entire equation. To compensate, we must add {3 \over 4}  

4

3

​  

 outside the parenthesis.

Therefore, the vertex \left( {h,k} \right)(h,k) is \left( {{1 \over 2},{{11} \over 4}} \right)(  

2

1

​  

,  

4

11

​  

).

Example 4: Find the vertex form of the quadratic function below.

y = 5x^2 + 15x - 5  

Solution:

Factor out 55 among the xx-terms. Identify the coefficient of the linear term inside the parenthesis which is 33. Divide it by 22 and square to get {9 \over 4}  

4

9

​  

.

Add {9 \over 4}  

4

9

​  

 inside the parenthesis. Since we factored out 55 in the first step, that means 5\left( {{9 \over 4}} \right) = {{45} \over 4}5(  

4

9

​  

)=  

4

45

​  

 is the number that we need to subtract to keep the equation unchanged.

Express the trinomial as a square of binomial, and combine the constants to get the final answer.

Therefore, the vertex \left( {h,k} \right)(h,k) is {{ - \,3} \over 2},{{ - \,65} \over 4}  

2

−3

​  

,  

4

−65

​  

.

Answer:

(x - 1 )^2 - 3

Step-by-step explanation:

( x - 1 )^2 + ( -3)

x^2 - 2x + 1 - 3

x^2 - 2x - 2

Represents the solution to the inequality -9=2/3x-7<5

Answers

Answer:

-3=x <13

Step-by-step explanation:

[tex] - 9 = \frac{2x}{3} - 7 < 5[/tex]

Multiply through by 3

[tex] - 27 = 2x - 21 < 15[/tex]

Add 21 to all sides

[tex] - 6 = 2x < 36[/tex]

Divide through by 2

[tex] - 3 = x < 18[/tex]

The solutin set is

[tex]{- 3 = x < 18}[/tex]

I need help asap!!!​

Answers

There are 360° total in a circle, so AB is half of the circle so it’s 180°. CBA is 180° also. 180°+55°=235°, 360-235= 125° which is AC

type in symbols to make 3,7,12,2 equal 45

Answers

Answer:

The answer is (3×7) + (12×2) .

[tex](3 \times 7) + (12 \times 2)[/tex]

[tex] = 21 + 24[/tex]

[tex] = 45[/tex]

PLEASE help me with this question! No nonsense answers please. This is really urgent.

Answers

Answer:

last option

Step-by-step explanation:

Let's call the original angle x° and the radius of the circle y. The area of the original sector would be x / 360 * πy². The new angle, which is a 40% increase from x, can be represented as 1.4x so the area of the new sector is 1.4x / 360 * πy². Now, to find the corresponding change, we can calculate 1.4x / 360 * πy² ÷  x / 360 * πy² = (1.4x / 360 * πy²) * (360 * πy² / x). 360 * πy² cancels out so we're left with 1.4x / x which becomes 1.4, signifying that the area of the sector increases by 40%.

3/4a−16=2/3a+14 PLEASE I NEED THIS QUICK and if you explain the steps that would be geat:) Thank youuuuuuu

Answers

Answer:

360

Step-by-step explanation:

3/4a - 16 = 2/3a + 14               ⇒ collect like terms 3/4a - 2/3a = 14 + 16               ⇒ bring the fractions to same denominator9/12a - 8/12a = 30                  ⇒ simplify fraction1/12a = 30                               ⇒ multiply both sides by 12a = 30*12a = 360                                   ⇒ answer

AB =
Round your answer to the nearest hundredth.
B
?
2
25°
С
A

Answers

Answer:

? = 4.73

Step-by-step explanation:

Since this is a right triangle we can use trig functions

sin theta = opp / hyp

sin 25 = 2 / ?

? sin 25 = 2

? = 2 / sin 25

? =4.732403166

To the nearest hundredth

? = 4.73

Other Questions
Find the vector and parametric equations for the line through the point P(0, 0, 5) and orthogonal to the plane 1x+3y3z=1. Vector Form: r After reading it write about whether or not you agree with the academic economic consensus that independent officials running the Federal Reserve are able to properly balance their dual mandate in a fair and balanced fashion with the needs of workers in one hand and the financial industry on the other. If you agree with the consensus view explain your reasons; or if you disagree and think that the officials are biased in favor of the financial industry explain your reasoning with some possible solutions to the problem. Write at least two paragraphs articulating your views. A corporate bond currently yields 8.5%. Municipal bonds with the same risk, maturity, and liquidity currently yield 5.5%. At what tax rate would investors be indifferent between the two bonds? What is the melting pot theory? Which equation represents the line that is perpendicular to y=3/4x+1 and passes through (-5,11)Will give brainliest!! Please answer this question now Human BonesName three components of bones and describetheir function. On May 10, 2017, Vaughn Co. enters into a contract to deliver a product to Greig Inc. on June 15, 2017. Greig agrees to pay the full contract price of $1,990 on July 15, 2017. The cost of the goods is $1,310. Vaughn delivers the product to Greig on June 15, 2017, and receives payment on July 15, 2017. Prepare the journal entries for Vaughn related to this contract. Either party may terminate the contract without compensation until one of the parties performs. What is an empirical formula? hich of the following is NOT one of the ways companies are using mobile apps? Group of answer choices track behavior across tablets and mobile devices utilize cookies to track mobile activity utilize GPS data to provide location-based offers track loyalty program participation add social value and entertainment to consumers' lives You own a farm and have several fields in which your livestock grazes. You need to order barbed-wire fencing for a small pasture that has a length of 5 yards and a width of 3 yards. The barbed wire must be long enough to be placed on all four sides of the outside pasture. How many yards of barbed-wire should you order? Which of the following systems has (1,1) as a solution? 1. 3x2y=1 5x+3y=8 2. 3x+2y=1 5x+3y=8 3. 3x+2y=1 5x3y=8 4. 2x+3y=1 5x3y=8 What is the width of the rectangle shown below? 4x + 3 A = 8x2 10x 12 What is the result when the number 90 is decreased by 10% Find the missing length. A. 60 B. 80 C. 64 D. 15 A large reflecting telescope has an objective mirror with a 14.0 m radius of curvature. What angular magnification in multiples does it produce when a 3.25 m focal length eyepiece is used? Combine the radicals. 224+554 A) 536 B) 56 C) 196 D) 936 Determine the convergence or divergence of the sequence with the given nth term. If the sequence converges, find its limit. (If the quantity diverges, enter DIVERGES.) an = 1/sqrt(n) Which is represented by the image? Two cards are drawn from a standard deck of cards.Part A: If they are drawn with replacement, what is the probability that both cards are 2s? Show your work. (1 point)Part B: If they are drawn without replacement, what is the probability that the first card is a club and the second card is a spade? Show your work (1 point)Part C: Which of the two scenarios in Part A or Part B represents dependent events? Explain your answer using complete sentences. Show your work. (2 points)